You are on page 1of 24

Paper - I : Physics - 1 to 50

Solutions

1.a. The power dissipated by a resistor is equal to i2 R. voltage of 120 V. The rms value of voltage is equal to the
Alternate forms of the equation which may be easier in peak voltage divided by the square root of 2.
certain cases are P = V2/R and P=IV. Here, since we know
V and some information pertaining to R, it is easiest to use Vpeak
P=V2/R. Vrms = ; Vpeak = Vrms 2
P= V2/R=(120) (120)/R 2
Substituting definition of resistivity, R=p1/A
P=(120) (120) A/pI The correct answer is 120 2 V, answer choice (a)
where A is the cross-sectional area of the wire, Πr 2 . 5.c. The current will be smallest when the resistance is largest.
We therefore want to rank the resistor combinations in
Plugging in values of A and I, with all distances in meters:
order of increasing resistance. The rule for combining
P= (120) 120) (3.14) (0.001( (0.001)/0.5p
P = (12)(12) (3) x 10-4/p resistors is that resistors in series add (R eq = R 1 + R 2 ),

PAGE
P=288(3) x 10-4/p
while resistors in parallel combine according to the equation
P = 900 x 10-4/p
P = 0.09/p (1 / R eq = 1 / R 1 + 1 / R 2 ). The important implication of
The closest answer is 0.1/pp. this rule is that two identical resistors in series have a total
2.d. The toast pops up when the magnetic force of the resistance of 2R, while two identical resistors in parallel
electromagnent pulling down on the tray becomes smaller have a total resistance of only 1/2R. The correct order is is
in magnitude than the spring force pushing up. Since we therefore #3 (total resistance R/3), # 4 (total resistance R),
want the toast to pop up sooner, we are looking for an #2 (total resistance 3R/2), #1 (total resistance 3R).
www.aieeepage.com
answer that either decreases the magnetic force or increases
the spring force. Answer (A) is incorect because decrease
6.a. Suppose that the frequency of the source is f.
the spring force (F = kx). Answer (B) is incorrect because
increasing the strength of the elecromagnent would increase Then from v= f λ
its magnetic pull and keep the toast tray down longer. The wavelengths of the longitudinal waves
Answer (C) is incorrect because decreaseing the separation
generated, in the two sections, should be such
of the capacitor plates will increase the capacitance
that,
(C = ε 0 A / d ). A larger capacitance means that the their phase difference, starting from the two ends
current through the electromagnet will not drop off as
quickly (time constant = RC, so larger C means longer
and reaching the middle one should be (2n + 1)π
time to charge). Answer (D) is correct because increasing
the resistance of the heating element actually decreases 1100ms −1
the power dissipated across the fixed voltage of 120 V. P where n ∈1 . Now, λH =
f
= V2/R. Here, V remains fixed at 120V. so if R gets larger,
the power gets smaller. With less power dissipated, the
310ms −1
toast will be lighter. (Note : A real toaster uses a high and λ0 =
resistance nichrome wire, and it is the resistor on the top f
branch that is adjusted to change the darkness of the toast
by altering the duration of the toasting process). A path difference of λ corresponds to a phase
3.c. We know that the ratio of the energy inputs to
compress the springs of the two toasters is 1 : 6. This 76
is the same as the ratio of the energies stored in the diffrerence of 2π . So, a length of path 2 cm
springs :

PAGE
38
1 would effect a phase of cm(2π ) radian
k1x12 λ
2 1
= ∴ The phase difference between the waves
1 Substituting x 2 = 2x1 :
k 2 x 22 6 reaching the middle diaphragm
2
2π  1  2π  1  1 1 
⇒  −   = πl  − 
1
2
k 1x12
1
k 1x12
k1 1
www.aieeepage.com λ0  2  λH  2   λ0 λH 

= 2 = = 
= π [76cm] 
f

f 
1 1
k 2 ( 2 x 1 ) 2 4k 2 6
−1 
k 2 x 22
−1
 310 ms 1100 ms 
2 2 For f to be minimum the phase difference should
be a ‘least’ odd multiple of π
k1 4 2
= = .
k2 6 3
 1 1 
4.a. The passage tells us that the wall outlet supplies an rms ∴π (76cm)  −  f =π
 310 × 10 cm / s 1100 × 10 cm / s 
2 2

PAGE - A Premier Institution for IIT - JEE & AIEEE at the National Level 1 of 24
∴ f = 568 Hz 11.c. No change in velocity imnplies no acceleration i.e.
no net force is acting on the proton, even under the
Q joint influence of electric and magnetic field. This
7.b. =2
q thing is possible under the following situations.
Situation A: E = 0, B = 0, i.e., no field exists
Q in the region.
⇒ q=
2 Situation B: E = 0, i.e. no electrostatic force.
i.e., the charge Q is divided equally into two parts B ≠ 0 , but the charge particle enters parallel to
and hence the force of repulsion between both of the field, so that net force equals to zero.
the parts is the maximum.
Situation C: E ≠ 0 , i.e., the charged particle
8.b. 1 T proton must experience an electrostatic force eE
Sincef = and hence must accelerate.
2l µ
Situation D: E ≠ 0 , B ≠ 0 and both
we have l2=kT (where k = constant)
shown in figure
or 2 log l = log k + log T

PAGE
Because in such a situation
1 1 E
or log l = log T + log k x x x x x x
2 2 p
which is of the form y = mx + C
1 x x x x x x
∴ Slope m = B
9.b.
2
www.aieeepage.com
When steady state is reached no current will flow F = evB (upwards) and m
in the branch having the capacitor. So Fe = eE (downwards)
and if both are equal in magnitude then even the
(2) (3) proton will suffer no change in its velocity and will
Rtotal = 2.8 +
2+3 continue to move along the dotted line as shown in

⇒ Rtotal = 4Ω E
figure. In such a situation, the velocity is v =
B
6
⇒ 1= = 1.5 A 12.a. For a fixed mass of gas, if two of the three
4 thermodynamic variables (P, V and T) coincide
corresponding to any two states, the third
automatically coincides, since they are bound by
 1/ 2 
Current in 2 Ω resistor = 12 =   1.5 the equation of state. Since internal energy is a state
1/ 2 +1/ 3  function, so the return of the gas to the initial state
implies no change in its internal energy.
3
⇒ I 2 =   (1.5)
5 13.a. hv=K.E. (T) + Work function (W)
⇒ I 2 = 0.9 A ⇒ hv= T + W
⇒ 4.25 eV = TA + WA (for Metal A)
Potential drop across 2.8 Ω is
⇒ 4.70 eV = TB + WB (for Metal B)

PAGE
V = (2.8) (1.5) Since TB = (TA-1.5) eV
V = 4.2 V
10.d. Absolute temperatures of the black body h
corresponding to curve P and Q are in the inverse also λ=
p
ratio of λm (Wein’s displacement law).
h  p2 
⇒ λ = ∵ = T = K .E .
TP 1987 2mT  2m 
i.e., =
TQ 2980 www.aieeepage.com
λ T
Area under curves represent the total power radiated ⇒ A
= B

λB TA
by a body and is proportional to the fourth power
of absolute temperature (|Stefan’s law)
1
4 Since λ A = λB ⇒ TA = 4TB
AP  TP  16 2
∴ =  =
AQ  TQ  81 ⇒ TB = TA − 1.50 gives

PAGE - A Premier Institution for IIT - JEE & AIEEE at the National Level 2 of 24
⇒ TB = 4TB − 1.5 RT  V
or = P0 3 − 
⇒ TB = 0.5eV V  V0 
⇒ TA = 2eV P0  V2
T = 3V − 
⇒ WA = 2.25eV R V0 
⇒ WB = 4.20 eV dT
14.d. Power developed = (heat capacity) rate of For T to be maximum =o
temperature rise
dV

 cal   0
C dT P0  2V  3
= (180 g )  0.1 0   0.5 ∴ = 3 −  = 0, or V = V0
 dV R  V0  2
 g − C  s 

d 2T −2 P0

PAGE
=9 cal/s 37.8 J/s
= = −ve, hence maximum.
P = τω dV 2 RV0
 180 × 2π 
∴ 37.8 = τ   or τ =2 N-m P0   3V0   3V0 2 1  9V0 P0
 60  ∴ Tmax = 3  −  ×  = 4R
15.c. Consider light to be incident at near normal
R   2   2  V0 
incidence. We wish to cause destructive interference
www.aieeepage.com
17.a. Electric field at some point P in the region between
between rays r1 and r2 so that maximum energy the spheres at a distance r from the centre
passes into the glass. A phase change of λ / 2 (according to Gauss’ las) is given by:
occurs in each ray for at both the upper and lower q
surfaces of the MgF2 film the light is reflected by a
medium of greater index of refraction. When striking
∫E P .ds =
ε0 +q +q
a medium of lower index of refraction, the light is +q
q a P
reflected with no phase change. Since in this or EP .4π r 2 = O
P’
porblem both rays 1 and 2 experience the same ε0 b
phase shift, no net change of phase is introduced c
by these two reflections. Hence, the only way a q
phase is change can occur, is if the two rays travel or EP =
through different optical path length is product of
4πε 0 r 2
the geometric path difference a ray travels through Similarly, electric field at point P outside the hollow
different media and the refractive index of the sphere is
medium in which it is travelling. For destructive
q
interference the two rays must be out of phase by EP ' =
an odd number of half wavelengths. Hence, the 4πε 0 r 2 ;
optical path difference needed for destructive
interference is: 1 q q q
Vinner =  a − b + c 
λ 4πε 0

PAGE
2 µ d = (2n + 1) , n = 0,1, 2....
2
1 q q q
Note that 2 µ d is the total optical path length that Vinner =  c − c + c 
4πε 0
the rays traverse when n=0.

λ / 2 λ 350 ×10−9 q 1 1
∴ d= = = ∴ Vab = Vinner − Vouter ;  − 
2µ 4µ 4 ×1.38 4πε 0  a b 

= 100nm = 1× 10 −1 m
www.aieeepage.com
18.b. ∵ wetting is complete, angle of contact θ=0
16.b. The equation of curve is, 2T Cos θ
Rise of liquid h=
P − 2 P0 P0 − 2 P0 rpg R O
=
V − V0 2V0 − V0
 2T 
or h = ∞ cos θ ∵ = cons tan t 
 −P   rpg 
P = (V − V0 )  0  + 2 P0
r
 V0 

PAGE - A Premier Institution for IIT - JEE & AIEEE at the National Level 3 of 24
A=amplitude]
nh cos θ
ie., = ⇒ cosθ = n
h cos θ g g gT 2
or A≥ = or A ≥
⇒ θ = cos −1 (n) ω 2  2π 2 4π 2
 
19.a. Velocity vector is perpendicular to magnetic field.  T 
Therefore, path of the particle is a circle of radius
mv0
r= , v = v0, as the speed of the particle F block
Bq
does not change in the magnetic field. Centre of
the circle is C.
CP = CQ x v0 x x

Piston
∴ ∠CPQ = ∠CQP P 90° x x x
V R x x
or 90 − α = 90 − β mg

PAGE
β
90° x x x
∴ α−β Q
x x x x Now, the maximum velocity vmax at that instant =
Further PQ = 2 PR ωA
= 2r cos (90 − α ) g gT
∴ vmax = or = 3.12 ms −1
= 2r sin α ω 2π
∴ PQ =
2mv0
sin α www.aieeepage.com
23.b. If there is no external electric field, then the charge
given to a conducting sphere gets uniformly
Bq distributed over its surface. Therefore, statement
∠PCR =∠RCQ =α (1) is correct.
If an external electric field exists, then the charge
∴ ∠PCQ =2α gets distributed over the surface of the sphere in
such a way that the electric field inside the sphere
Arc PSQ = (2π − 2α )r
can become equal to zero. Hence, distribution of
2mv0 (π − α ) the charge on the surface of sphere will be non-
= uniform. Therefore, (2) is correct. Obvisously (3) is
Bq wrong.
Since electric field inside the conducting sphere is
PSQ 2m(π − α )
∴ t PSQ = = equal to zero, therefore, potential difference
v0 Bq between two points in the sphere is equal to zero. It
means the potential is same at every point of the
5 sphere. Therefore, (4) is correct.
20.c. Volume of body = 3
=1×10-3 m 3 24.b. The gravitational field intensity at the centre of the
5×10
solid sphere due to the ring is given by
∴ Force of upthrust = wt. of water displaced
= 1×10−3 m3 × 1000 kg / m3 = 1 kg GM 1d
E=
∴ Reading of balance A = 10 kg + upthrust (d + h 2 )3/ 2
2

PAGE
= 10 + 1 = 11 kg
and reading of balance B = 5 kg - upthrust R
= 5 - 1 = 4 kg.
21.a. At u = f ,v = ∞ 2 2R = d

At u=0 (i.e., object is at pole) R θ


v=0
www.aieeepage.com
(image is also at pole) O
Satisfying these two conditions only option (a) is
correct.
22.b. The situation, when the block is just below the
mean position is , the restoring forces acting on GM 1 (2 2 R) GM 1 (2 2 R)
the piston, causes a normal reaction F to act on the = =
(8R 2 + R 2 )3/ 2 27 R 3
block. For the block to separate.
The toral mass of the solid sphere can be assumed
F ≥ mg to be concentrated at its centre 0.
i.e., mω 2 A ≥ mg [where ω = ang. and ∴ Force acting between M1 and M2 is F=EM2
25.a.

PAGE - A Premier Institution for IIT - JEE & AIEEE at the National Level 4 of 24
26.a. To locate the instantaneous centre of rotation of a
 dq1   dq2 
body, at least two points on it should be choosen,
∴  ≠ 
whose direction of velocity is known. Lines should  dt   dt 
be drawn perpendicular to the direction of velocity ∴ statement (1) is wrong
from those points. The point of intersection gives 30.b. If V1 and V2 be the velocities of A and B respectively
the instantaneous centre of rotation. In this case, after collision, then
points A and B move respectively along X and Y m(10) = mv1+mv2
axes. So lines should be drawn from point A and B,
⇒ v2-v1 = 10 (i)
perpendicular to X and Y axis. They intersect in the
If e be the minimum coefficient of restitution, then
first quadrant.
27.a. Chromatic aberration is a defect due to which the v2 − v1 ≥ e(10)
images of different colurs of a point object are
formed at different places. ⇒ v2 − v1 ≥ 10 e (ii)
Since refractive index of a matterial depends not
only upon that material but also upon the colour of
Adding (i) and (ii) v2 ≥ 5(1 + e) (iii)
By conservation of energy, for ball B

PAGE
incident rays, therefore, when a light ray is refracted
through a material, then the images of different 1 2
colours may be formed at different place only when m(10) (3.2) = mv2
refraction takes place. Hence, the mirror cannot give 2
rise to chromatic aberration. It means statement (a)
and (2) are correct.
⇒ v22 = 64
Spherical aberration is a defect due to curvature of Substituting for v2 from (iv) in (iii) ...(iv)
the refracting or reflecting surface. It can be
www.aieeepage.com
produced by mirror as well as lens. Hence (3) and
(4) are also correct.
64 ≥ 25(1 + e)2
8
⇒ (1 + e) ≤
1 1  hR 
2 5
28.b. v = π r 2h = π   h
3 3 H  3
⇒ e≤
5
dV π R 2 dh
∴ = 2 ( h) 2 = K ( say ) 31.c. When the ball is pressed down, its c.g. gets lowered
dt H dt hence, its P.E. decreases. However, the water
displaced by the ball rises and hence its P.E.
dh H 2 K increases. The net increase in P.E. will be equal to
∴ = dV the work done.
dt π R 2 h 2 R
= k
dt 1
2
Since, the sp. gravity of the ball is 2 , so half its
 3 dh  3H K
or   = r volume will remain immersed initially.
 4 dt  4π R h
2 2
H Increase in P.E. of water

C R
4 3  2 3   3R 
or y= ⇒ log y = log C C.G
− 2 log x  π R ρ  Rg −  π R ρ   g
x2 3  3  8 
Which is of the form Y = -2x +C
 3  26π R ρ g
4
∴ Slope = -2 2

PAGE
= π R4 ρ g 2 −  =
29.a. τ c1 = τ c2 = 2CR = τ c ( say ) 3  8 3×8
Decrease in P.E. of ball =
q1 = ( EC )[1 − e − t / τ c ]
4 3 ρ  2
 πR  Rg = π R ρ g
4
and q2 = (2 EC )[1 − e − t / τ c ] 3 2 3
q1 1 ∴ Net work done = net increase in P.E.
= ( at any time)
q2 2 www.aieeepage.com
=
 26 2  5
− π R ρg = π R ρg 4 4
 
∴ The ratio of steady state charges is also 1:2  24 3 12
dq1 EC − t /τ c
= e 1 1 1 
dt τc 32.a. = ( µ − 1)  + 
f  R1 R2 
dq2 2 EC − t / τ c
and = e 1 1 2
dt τc or = × or r = f
f 2 R

PAGE - A Premier Institution for IIT - JEE & AIEEE at the National Level 5 of 24
1 1 1 1 1 2
∴ = + = + =
F f1 f 2 f f f ∴ R=
ax 2
2 +1
= ax 2 ( )
2 − 1 = ax (2 − 2)

∴ F = ( f / 2)
F 1
∴ Focal length of the combination 35.a. Given u = 5 i + 2 j ; a = = 2i − j
m 2
f R
= = = half the radius of curvature
2 2 If v be the vlocity at time t, then from v = u + at
33.c.  1 
ω0 ⇒ v = (5 + 2t )i +  2- t  j
ω0  2 
When motion is purely along X-axis, Y component
V V of velocity should be zero
N E
1

PAGE
∴ 2− t = 0 ; ⇒ t=4s
2
Train Q Train P 1
Using s = ut + a t2
Evidently, the track of tain P as well as Q rotates 2
with earth, due to its spin motion, (say with an
1
( r2 − r1 )
2
angular velocity ω 0 ). If R be the equatorial radius, = ut + at 2
www.aieeepage.com
then, the linear speeds of trains P and Q, relative to
2
If ( x iˆ + yiˆ) be the position vector after time t,
their respective tracks will be (v+ ω 0 R) and (v-
then
ω 0 R)  1
( x iˆ + y iˆ) − (2 iˆ + 3 iˆ) = (5 iˆ + 2 ˆj ) 4 +  iˆ − ˆj  (16)
If T1 and T2 be their respective forces (centripetal)  4 
offered on the ground,
Then 36 iˆ + 4 ˆj
m m
T1 = (v + ω 0 R) 2 and T2 = (v − ω 0 R ) 2 ∴ x iˆ + yiˆ = 38 iˆ + 7 ˆj
R R 36.c.
Evidently T1 > T2
34.a.
12 ax
A B A B
12 1 1
ax ax

D a 2x C D C
ax

PAGE
(i) (ii) +π /2 Q Q Qdθ
E= ∫ dE , dq = dl = Rdθ =
Current entering at A will divide into I1 along AC −π / 2 πR πR π
and I2 each in ABC and ADC since the resistances
π /2 1 dq
ABC and ADC are equal. E= ∫ cos θ
Now, the resistance AB=resistance AD. −π / 2 4πε 0 R 2
Hence, the potentials of B and D are equal. Hence,
there will be no current in BD. The circuit is 1 Q cos θ dθ 1 2Q
www.aieeepage.com
π /2
equivalent to one shown in figure (ii). If R is the =∫ =
4πε −π / 2
0 πR 2
4πε 0 π R 2
equilent resistance.
1 1 1 1 37.b. The component of velocity of the ball normal to
= + +
R 2ax 2ax 2ax the plane is -20 m/s. (before striking)
Since the collision is elastic, immediately after the
1  1  1  2 +1 collision the component of velocity, of the ball
=  1+ =   normal to the plane bcomes +20 m/s.
ax  2  ax  2 

PAGE - A Premier Institution for IIT - JEE & AIEEE at the National Level 6 of 24
∴ Initial velocity of the ball immediately after it
strikes the plane is
or tan θ = 2

u = 10iˆ + 30 ˆj + 2 − kˆ ∴ cos θ =
1
=
1

( 2 ) +1 3
2
∴ Using v = u + at 2

Final velocity after t= 2 sec will be


40.b. Let I1 and I2 be currents through B1 and B2.
v = (10iˆ + 30 ˆj + 20kˆ ) − 10kˆ(2) (∵ a = − gkˆ )
Then I1 × R 2 + X C2 = 220
= 10iˆ + 30 ˆj
38.b. Let 2a be the side of the traingle and b the length R 2 + X C2 R 2 + (1/ ω C ) 2
I2
AE.
∴ = =
I1 R 2 + X L2 R 2 + (ω L) 2
AH GH
=
AE EC
R 2 + 40

PAGE
x x x x x x =
 AH  R 2 + 9.87
∴ GH=   EC x x x A x x x
 Ae 
x x x x x x So I 2 > I1
(b − vt ) Bulb B2 will be brighter than B1. As frequency
= .a x xF x
H
x Gx x
vt
b increases, XC decreases while XL increases, so I2
D E C becomes less than I1 . Hence brightness of B1 will
a
= a −   vt
b
www.aieeepage.com
41.c.
increases and that of B2 will decrease.
Initial velocity of P and Q are :

u1 = 10 (iˆ + ˆj )
 a 
∴ FG=2GH=2  a- vt 
 b  and u2 = 20 (−iˆ + ˆj )
Induced e.m.f. Initial position vectors of P and Q are :
e = Bv(FG)
s1 = 20 ˆj; s2 = 20 iˆ
 a  Their position vectors after time ‘t’ will be
= 2bv  a − vt 
 b 
s1 = 20 ˆj + 10 (iˆ + ˆj )t − 5t 2 ˆj
∴ Induced current
e 2 Bv  a  and s2 = 20 iˆ + 20 (-iˆ + ˆj )t − 5t 2 ˆj
I= =  a − vt 
R R  b  ∴ Their relative position vector will be
s '1 − s '2 = (−20 + 30t ) iˆ + (20 − 10t ) ˆj
or I = k1 − k2t
Thus, I - t graph is a straight line with negative ∴ s '1 − s '2 = 10 (3t − 2)2 + (2 − t )2
slope and positive intercept
39.d. Let ux and u y be the velocity of projection, = 10 (9t 2 + 4 − 12t + 4 + t 2 − 4t )

PAGE
along the horizontal and vertical then 2
i.e., s '1 − s '2 = 100 (10t 2 − 16t+8
uy
tan θ = (i) For maxmum or minimum separation
ux
d 2
At a height equal to three - fourth that of maximum s '1 − s '2 = 0
dt
height, the vertical component of velocity ( v y ) will
www.aieeepage.com
= 100 [20t - 16] = 0
be = t = 4/5 s

 u y2  3 u d 2
v = u − 2g 
2 2
⇒ vy = y s '1 − s '2 = 2000 = +ve;
y y 
 2g  4 dt 2

  2
Hence the separation is minimum
vy uy 4
Given tan (900 − θ ) = = (ii) Putting t = s in equation (i )
ux 2u x 5
From (i) and (ii) tan θ = 2cot θ

PAGE - A Premier Institution for IIT - JEE & AIEEE at the National Level 7 of 24
which both forces act is the same, then the absolute value
of the work done by the normal force is greater than the
 16  4
s '1 − s '2 = 10 10   − 16   + 8 absolute value of the work done by gravity.
 25  5 Answer choice (B) is not true because in this case the
normal force is NOT perpendicular to the direction of
motion. Answer choices (C) and (D) follow from the
2
= 20 m misconception that just because an object is moving down,
5 the net force is down. This is not ture. The direction of the
net force is the sae as the direction of the change in velocity,
42.b. Any transition causing a photon to be emitted in not of the velocity itself.
the Balmner series must end at n=2. This must be 49.b. A pulley system with one moveable pulley has a
followed by the transition from n=2 to n = 1, mechanical advantage of 2. (Note that fixed pulleys do not
emitting a photon of energy 10.2 eV, which increase the mechanical advantage.) Therefore, if the weight
corresponds to a wavelength of about 122 nm. This of the load to be lifted were W, an applied force of only W/
belongs to the Lyman series. 2 would be necessary in an ideal, friction-free system:
43.c.
force on load
MA =

PAGE
44.b. Using RHSR v × B is along east applied force
But F = −e (v × B ) [ ∴ electrons carry -ve
charge] W W
2= →x=
x 2
∴ Direction of F is opposite to that of (v × B ) .
Remember that if the load is to move at a constant speed,
45.b all the forces on the laod must balance.

www.aieeepage.com
46.c. The upper limit on mechanical advantage will be the Because any real system is NOT friction-free, some energy
mechanical advantage of a friction free system. In this will be converted to heat. When the load is being lowered,
case, a free body diagram of the boxed portion of the system this energy loss would subtract from the gain in kinetic
reveals that six rope segments actually support the load. energy experienced by the load. In other owrds, friction is
Therefore, each segment need only support 1/6 the weight doing negative work working in conjunction with the
of the load. The tension in the rope created by the applied applied force to hold the load back against gravity. The
force is one sixth the weight of the load and the upper limit applied force would therefore be less than W/2. When the
on mechanical advantage is therefore 6. Answer choice load is being raised, the energy lost to friction is again
(A) is simply a count of the moveable pulleys, but it must subtracting from the gain in kinetic energy of the load,
be remembered that each moveable pulley gives a factor of holding the load back. But this time, this effect is working
TWO in mechanical advantage. Choice (D) arises from a in conjunction with gravity and in the opposite direction
rope segment count that includes the segment being pulled. as the applied force. To counterbalance the effect of friction
However, this segment does not support the load, as do and raise the load at a constant speed, the applied force
the other six. It is therefore not included in calculations of would therefore need to be greater than W/2.
mechanical advantage Comparing these two situations, wer see that the applied
47.b. : If 20% of the energy is lost, then 80% of it is retained as force necessary when the load is being lowered is less than
useful work in lifting the load. The efficiency can then be the applied force necessary when the load is being raised.
calculated: for each unit of work input, 0.8 units of work 50.b. The pulley on the flagpole is a FIXED pulley. According
are done on the load: to the passage, fixed pulleys do not contribute to
mechanical advantage, but merely redirect the rope.
work output 0.8 Therefore, this system does not change the amount of
efficiency = = 0.8
work input 1 force needed to lift the object. The applied force is the
same as the force on the load, and the mechanical advantage

PAGE
The efficiency equation can be re-arranged as: (force is 1.
applied) (distance of applied force) (efficience) - (force on
load) (distance load travels)

(1000 N) 1
force applied =
(0.8) 6
48.a. In elevator problems, it is usually critical to determine the

www.aieeepage.com
direction of the acceleration and the direction of the net
force and then relate this information to the relative sizes
of the forces in a free-body diagram.
If the elevator isw moving down and slowing down, then
the direction of the net acceleration is up. The direction of
acceleration is the same as the direction of the net force.
Therefore, the net forece also points upward. The only
two forces that act on the person are gravity and the normal
force. If the net force is up, then the normal force must be
greater than the gravity force.
Since work is force times distance, and the distance over

PAGE - A Premier Institution for IIT - JEE & AIEEE at the National Level 8 of 24
Paper - I I : Mathematics - 51 to 100
Solutions

51.a. He will be left with no money at the of end of the


tenth round I and only I he gets a tail on each of = y ( − cot x ± cosec x)
them. The probability of this = q10 (q = 1 − p) dy
Thus, we have = (− cot x + cosec x )dx
[Note that the player cannot drained out of his dx
money before 10th round]
52.a. Again he cannot be drained at the 13th round I A x
⇒ log y = − log sin x + log tan + log c
gets at least 2 heads in the firist 10 rounds. To 2
finish at the 14th round, A must get exactly 2 heads
x
in the first 10 rounds and a tail on all the rounds c tan
2 = c c
from 11th to 14th. This has a probablity ⇒y= =
sin x x 1 + cos x
10 2 cos 2
C 2 p 2 q12 .

PAGE
2
Therefore required probability =
dy
q10 +10 C1 pq11 +10 C 2 p 2 q12 Solving y = −(cot x + cosec x)dx,

= q10 (1 + 10pq + 45p2 q 2 )


e
53.a. Here, P be (x, y, z) we get y =
1− cos x
then,
www.aieeepage.com
x = r sin θ. cos φ, y = r sin θ sin φ, z = r cos θa
⇒ x = 2 sin −1 C
2y
⇒ 1 = r sin θ. cos φ, 2 = r sin φ, z = r cos θ

⇒ 12 + 22 + 32 ∵ f ' ( x) =
1
=
1 1  x
= sec 2  
55.c. 1 + cos x 2 cos 2 ( x / 2) 2 2
= r 2 sin 2 θcos 2φ + r 2 sin 2 θ sin 2 φ + r 2 cos 2 θ
Integrating both sides with respect to x, we have
x
P(1, 2, 3) f ( x) = tan   + c
2
θ
x
O
∴ f (0) = 0 + c = 3 then f ( x) = tan   + 3
2
φ
π π
M ∴ f   = tan   + 3 = 4
2
  4

= r 2 sin 2 θ (cos 2 φ + sin 2 φ) + r 2 cos 2 θ π 22 11 53


Now, 3 + = 3+ = 3+ = ≈ 3.78
4 7× 4 14 14
= r 2 sin 2 θ + r 2 cos 2 θ = r 2
π 22 11 32
⇒ r = ± 14 and 3 + = 3+ = 3+ = ≈ 4.57

PAGE
2 14 7 7
∴ from (i), we have ∵ 3.78 < 4 < 4.57
1 2 3 π π π
sin θ cos φ = + , sin θ sin φ = , cos θ = Hence 3 + < f   < 3+
14 14 14 4 2
  2
(neglecting -ve sign assuming acute angles) It cal also be checked that other options do not
satisfy the conditions.
sin θ sin φ 2
www.aieeepage.com
∴ = sin θ 5
tan θ = =
sin θ cos φ 1 and The given curve is y = be x/a
cos θ 3 56.a.

5 Let us consider a point ( x1 , y1 ) on the curve.


⇒ tan φ = 2 and ⇒ tan θ =
3 Then y1 = be x1 / a
dy
54.c. Solving for , we obtain dIferentiating the curve y = be x / a with respect
dx
to we get
dy −2 y cot x ± 4 y cot x + 4 y2 2 2
=
dx 2

PAGE - A Premier Institution for IIT - JEE & AIEEE at the National Level 9 of 24
I conic be an ellipse, then SO + S’O = 2a
dy 1 and SS’ = 2ae
= be x / a .
dx a
SS' 386
∴e = =
 dy  b S'O + SO 38
  = e x1 / a
∴  dx ( x1 , y1 ) a I conic be a hyperbola, then S' O - SO = 2a and
SS’ = 2ae

 dx  ∴e =
SS'
=
386
Thus, the length of subtangent = y1   S' O + SO 12
 dy ( x1 , y1
59.a. let PA and PB be the tangent from P( x1 , y1 ) to
a a the given circle with centre C(-g, -f) such that
= y1 = be x1 / a . x1 / a = a (constant) [using (i) and (ii)]
be x1 / a be θ
∠APB = θ then ∠APC = ∠APC = ∠CPB =
⇒ Subtangent is of constant length a 2

PAGE
From right angled triangle PAC.
 dy 
again, length of subnormal = y1   θ CA g2 + f 2 −c
 dx  x1 y1 tan = =
2 PA S1

be x1 / a
= be x1 / a g2 + f 2 −c θ S1
a ⇒ θ = 2 tan −1 and cot =
S1 2
www.aieeepage.com
g2 + f 2 −c
1 1 Also,
= (be x1 / a ) 2 = y 2 [using (i)]
a a 1 - tan 2 θ/2 S1 − ( g 2 + f 2 − c ) S1 + c − g 2 − f 2
Therefore, subnormal varies as the square of cos θ = = =
1 + tan 2 θ/2 S1 + ( g 2 + f 2 − c ) S1 − c + g 2 + f 2
ordinate.
57.a. Case I : I n is even, say n = 2k. Then limit is
1 1 − tan 2 θ/2 2 S1
cot θ = = ≠
−6k + (−1) 2k
−6 k + 1 tan θ 2 tan θ/2 g + f 2 −c
2
lim = lim
k →∞ 8k − ( −1) 2 k k →∞ 8k − 1

A
1
−6k +
= lim k = −6 = −3
(x1 , y1 )
k →∞ 1 8 4 θ/2 C(-g, - f)
8− P θ/2
k

1 B
{as → 0, where k → ∞}
k
60.b. Let the sides of a triangle be a, ar, ar 2
Case II : n is odd, say n = 2k + 1. Then limit is
∵ ar 2 is the greated side (r > 1) ∴ a + ar > ar 2
−3(2k + 1) − (−1) 2 k +1

PAGE
−6k − 3 + 1
lim +
= lim 1− 5 1+ 5
k →∞ 4(2k + 1) − ( −1) 2 k 1 k →∞ 8k + 4 + 1
∵ r 2 − r −1 < 0 ⇒ <r<
2 2
2
−6 − 1+ 5
= lim k = −3 ⇒1< r <
2
k →∞ 5 4
8+ ∴ (1) is correct

www.aieeepage.com
k
1 1 2
Also r < (6 + 2 5 ) = (3 + 5 ) and
−3n + (−1) −3 n 4 2
∴ lim = , n even or odd
n →∞ 4n − ( −1) n
4 1 1
r4 < (14 + 6 5 ) = (7 + 3 5 )
58.b. I two foci be S (5, 12) and S’(24, 7) and it passes 4 2
through origin O. 1 1
2 4
∴ 1+ r − r < 1+ (3 + 5 ) − (7 + 3 5 ) = −1 − 5 < r
Then So = 24 + 144 = 13; S ' O = 576 + 49 = 25 2 2

and SS’ = 386 a2 + a2 r 2 − a2r 4 1+ r2 − r 4 1


cos C = = <
2a 2 r 2r 2

PAGE - A Premier Institution for IIT - JEE & AIEEE at the National Level 10 of 24
π π sin 2 A + sin 2 B = 1 + cos C
cos C < cos ⇒C >
3 3 62.b. Note that
∴ (4) is correct P( Z ≤ m) = P{X ≤ m, Y ≤ m} = P{X ≤ m} P{Y ≤ m}
Also But
a 2 + a 2r 4 − a 2r 2 1+ r 4 − r 2 1 2 1  P{Y ≤ m} = P{X ≤ m} = P(X = 0) + P(X = 1) + ........ + P(X = m)
cos B = = =  r + 2 − 1
2a 2 r 2 2r 2 2 r  p(1 − q m )
= p + pq + pq 2 + ..... + pq m −1 = = 1− qm
1− q
1   1
2
1
=  r −  + 1 >
2  2  2 ∴ P(Z ≤ m) = (1 − q m ) 2
Now, {(Z = m) P ( Z ≤ m) − P ( Z ≤ m − 1)
π π
∴ cos B > cos ⇒ B <
3 3 = (1 − q m ) 2 − (1 − q m −1 ) 2
π = 1 − 2q m + q 2m − (1 − 2q m −1 + q 2m − 2 )

PAGE
2
Also, a < ar < ar ⇒ A < B < C ⇒ A < B < <C
3
= 2pq m-1 − p(1 + q)q 2m − 2
Hence (2) is also correct and (3) is incorrect.
Clearly,
61. a. sin 2 A + sin 2 B + sin 2 C
2p p(1 + q)
1 1 ∑ [2pq m −1 − p(1 + q)q 2m − 2 ] = − = 2 −1 = 1
= (1 − cos 2 A) + (1 − cos 2 B ) + sin 2 C m ≥1 1− q 1− q2
2 2
63.b. We note that eacch option is of the form
1
= 1 − (cos 2 A + cos 2 B) + sin 2 C
2
www.aieeepage.com
(n + 1) > or < n n n +1

n
1 (n + 1) n 1  1
= 1 − {2 cos( A + B ). cos( A − B)} + sin 2 C Now, = 1+
2 n n +1 n  n 
= 1 + cos C. cos(A - B) + 1 - cos 2C But
n 2 3
= 2 − cos 2 C + cos C. cos (A - B)  1 1 n(n + 1)  1  n(n + 1)(n + 2)  1 
1 + n  = 1 + n. n + 2!  n  + 3!
  + ....
    n
∴ sin 2 A + sin 2 B + sin 2 C ≤ 2 − cos 2 C + cos C
1  1  1  1  2 
{∵ the greatest value of cos (A - B) = 1} = 1+1+ 1 −  + 1 − 1 −  + .....
2!  n  3!  n  n 
= 2 − (cos2 C − cos C)
n
 1
 1 1 9  1
2 Clearly, 1 +  > 2
= 2 −  cos 2 C − cos +  + = −  cos C −   n
 4  4 4  2
n
 1 1 1 1
9 Also 1 +  < 2 + + 2 + 3 + ....∞
∴ sin 2 A + sin 2 B + sin 2 C ≤  n  2 2 2
4
Now, for positive quantities, AM ≥ GM 1

PAGE
sin 2 A + sin 2 B + sin 2 C 3 = 2 + 2 = 3, provided n > 3
∴ ≥ sin 2 A sin 2 B sin 2 C 1−
1
3 2
∴ from (2),
n
 1
9 ∴ 2 < 1 +  < 3
2 2 2  n
4 ≥ sin A + sin B + sin C ≥ (sin A sin B sin C ) 2 / 3
3

3
3
www.aieeepage.com
2 1 1
⇒ < 1 + 
n n n
n
<
3
n
< 1, for n > 3
∴ ≥ (sin A sin B sin C ) 2 / 3
4
(n + 1) n
3 ∴ < 1 ⇒ (n + 1) n < n n +1 for n > 3,
 3 2 3 3 n n +1
or sin A sin B sin C ≤   , i.e., which is satisfied only by the option (2) and (3)
4
  8
64.a. (i) xy > 0, 3x 2 − xy − 2 y 2 = −7 or
Also in ∆ ABC, sinA > 0, sinB > 0, sin C > 0
and from (1) (3 x + 2 y ) ( x − y ) = −7

PAGE - A Premier Institution for IIT - JEE & AIEEE at the National Level 11 of 24
x and y being integers, we can trake 3x + 2y = 7 ⇒ p>4
and x - y = - 1 which given x = 1, y = 2. I x and
67.c. To get a triangle , we either take two points on AB
y are charged to -x, -y the equation remains the
and ne pt. on AC or one pt. on AB and two
same so that x = -1, y = -2 is also a
pts. on AC
solution pair. It is easily seen that
∴ no. of triangles formed
3x + 2y = - 1 and x = y = 7 do not given
integral solutions. = ( m C 2 ) ( n C1 ) + ( m C1 ) ( n C 2 )
(ii) xy = 0 will not lead to integral solutions.
(iii) xy < 0. The equation is m(m − 1) n(n − 1)
= .n + m.
2 2
3x 2 + xy − 2 y 2 = −7 or (3x - 2y) (x + y) = - 7
1
3x - 2y = -7 and x + y = 1 leads to x = - 1, y = 2. = mn (m − 1 + n − 1)
As before x = 1, y = - 2 is also a solution set. 2
The required complex numbers are 1 + 2i, 1 - 2i, 1
-1 + 2i, -1-2i which form two conjugate pairs. = mn (m + n − 2)
2
Discriminant D = 4p 2 − 4(p 2 − 1) = 4 > 0

PAGE
65.b.
68.b. ∆ = ( 13 + 3 ) [25 − 5 2 ]
∵ Roots of the equation are real and distinct
Now both the roots are less than − 2 5 (5 15 + 25 − 3 10 − 65 10 )

-2p + 5 ( 15 5 + 5 5 − 15 − 5 65 )
4 if D ≥ 0, f(4) > 0 and 4 > -
2
= 25 13 + 25 3 − 65 2 − 5 6
⇒ 16 − 8p + p 2 − 1 > 0 and 4 > p
⇒ (p − 3) (p − 5) > 0 and p < 4 www.aieeepage.com
− 50 3 − 50 5 + 30 2 + 10 5 26

⇒ p < 3 or p < 5 and p < 4 ⇒ p ∈ (−∞, 3) + 5 15 + 25 − 15 5 − 25 13


Again both the roots are greater than -2 I = −25 3 − 35 2 − 65 5 − 5 6 + 10 5 26
-2p
D ≥ 0, f( −2) > 0 and - 2 < - + 5 15 + 25
2
= A real irrational number.
⇒ (4 + 4 p + p 2 + 1) > 0 and 3 < p 69.c. Let f(x) = x tan x
⇒ (p + 3)( p + 1) > 0 and p > -3 ∴ f ' ( x) = x sec2 x + tan x
⇒ p < −3 or p > −1 and p > -3 ⇒ p ∈ (-1, ∞) x + sin x cos x
= >0 [∵ 0 < x < π/2]
Further exactly one root lies in the interval cos 2 x
(-2, 4) I D > 0 and f(-2) f(4) < 0
∴ f (x) is increasing function.
⇒ ( p + 3)( p + 1)( p − 3)( p − 5) < 0
Now x1 < x2
⇒ p ∈ (−3,−1) ∪ (3, 5)
⇒ f ( x1 ) < f ( x2 )
Finally, 1 lies between the roots I D > 0 and f(1) < 0
∴ x1 tan x1 < x2 tan x 2
⇒ 1− 2 p + p2 −1 < 0
⇒ p ( p − 2) < 0 ⇒ 0 < p < 2 ⇒ p ∈ (0, 2) x1 tan x2 tan x2 x1
⇒ < ⇒ >

PAGE
x2 tan x1 ; tan x1 x2
66.b. | α -β | > 3p
70.d. Since f(x) is defined for +ve as well as -ve value
2
I α, β are the roots of x + px + 1 = 0 of x and f(x) ≥ 0 ∀ x ∈ D f
so that α + β = -p, α β = 1 Hence the graph of f(x) will lie on right as well
as on left y-axis and above the x-axis.
⇒ (α - β) 2 > 3 p ⇒ (α + β) 2 − 4αβ > 3p ∴ graph of f(x) lies in Ist and IInd quadrant.
2
⇒ (− p) − 4.1 > 3p www.aieeepage.com
2
⇒ p − 3p - 4 > 0
71.a. Let I = ∫
xe x
dx ; Put 1 + e x = t 2
⇒ ( p − 4) ( p + 1) > 0 1+ ex
⇒ p > 4, p > −1 or p < 4, p < −1 ∴e x dx = 2t dt
⇒ p > 4 or p < -1
log(t 2 − 1).2t dt
But p is not -ve Also x = log(t 2 − 1) ;∴ I = ∫ t
[∵ If p is - ve, then 3p is not real]


= 2 log(t 2 − 1)dt ; ∫
= 2 log(t 2 − 1) . 1 dt
I II

PAGE - A Premier Institution for IIT - JEE & AIEEE at the National Level 12 of 24
 2t  1
 ∫
= 2 log(t 2 − 1).t − 2
t −1
.t dt 

⇒ cot A cot B cot C ≤
3 3

2t 1
= 2t log(t 2 − 1) − 4 ∫t 2
−1
.t dt ⇒ K≤
3 3

t 2 −1 +1 74.d. 2 sin 2 θ - 3 sin θ - 2 = 0


= 2t log(t 2 − 1) − 4 ∫ t 2 −1
dt
⇒ (2 sin θ + 1) (sinθ - 2) = 0
 t −1  1
= 2t log (t 2 − 1) − 4t − 4 log +c ⇒ sin θ = - [∵ sin θ - 2 = 0 is not possible]
 t +1 2

= 2 1 + e x .x − 4 1 + e x  π 7π
= sin  −  = sin
 6 6
1  1 + e x −1 
− 4. log +c

PAGE
   π 7π
2
 1 + e x
+ 1  ∴ θ = nπ + (−1) n  −  = nπ + (-1) n
 6 6

1+ ex −1 n 7π
= 2( x − 2) 1 + e x − 2 log +c Thus, θ = nπ + (-1)
1+ ex +1 6
75.b. Let
x
1 + e −1
∴ f ( x) = 1 + e x , g ( x) =
1+ ex +1 www.aieeepage.com c = c1iˆ + c 2 ˆj + c3 kˆ

(c iˆ + c ˆj + c kˆ). (iˆ + ˆj )
∴ c12 + c22 + c32 =| iˆ + ˆj |2 = 2

1 2 3
2a
Also,
2a − x x 5/ 2 2
72.b. Area of the region = ∫
0
a2
dx
c1iˆ + c2 ˆj + c3 kˆ). (iˆ + kˆ) 1
= =
2 2
Put x = 2a sin 2 θ ∴ dx = 4a sin θ cosθ dθ
⇒ c1 + c2 = c2 + c3 = 1 ⇒ c1 = c3 and c 2 = 1 − c3
π/2
2a cos θ. (2a sin θ) 2 5/2
4a sin θ cos θ dθ Hence from (1),
= ∫ a 2
1 1
0 3c32 − 2c3 − 1 = 0 ⇒ c3 = − or 1 when c 3 = −
3 3
π/2


= (8) (4) sin 6 θ cos 2 θ dθ
0
1
c = − (iˆ − 4 ˆj + kˆ). When c 3 = 1, c = iˆ + kˆ
3
π/4
5.3.1.1 π 5π 2
= 32a 2 . . =
8.6.4.2 2 8
a 76.a. ∵ In = ∫0
tan n x dx

Also area of circle = πa 2 π/4

5π 2 5
= ∫ 0
tan n - 2 x(sec 2 x − 1)dx

∴ reqd. ratio = a : π a2 = = 5 : 8.

PAGE
π/4 π/4
∫ ∫
8 8 = tan n - 2 x sec 2 x dx − tan n - 2 x dx
73.a. We know tht I A + B + C = π, then 0 n

tan A + tan B + tan C = tan A tan B tan C ...(1) π/4


 tan n −1 x 
Since A.M. ≥ G.M. In =   − I n −2
tan A + tan B + tan C  (n − 1)  0
∴ ≥ (tan A tan B tan C)1/ 3
3 1
www.aieeepage.com
⇒ I n + I n−2 = ⇒ (1) is correct
⇒ tanA + tanB + tanC ≥ 3(tanA tanB tanC) 1/3 n −1
Replace n by (n + 1)
⇒ tanA tanB tanC ≥ 3(tanA tanB tanC)1/3 (By(1))
1
⇒ ( tanA tanB tanC)3 ≥ 27(tanA tanB tanC) Then I n +1 + I n −1 = ⇒ (2) is correct
n
⇒ ( tanA tanB tanC)2 ≥ 27  π
Now, in interval 0, tan n x < tan n − 2 x
 4 
⇒ tanA tanB tanC ≥ 3 3

PAGE - A Premier Institution for IIT - JEE & AIEEE at the National Level 13 of 24
π/4 π/4
F " f " g " 2c
∴ ∫0
tan n x dx < ∫ 0
tan n - 2 x dx
or
F
=
f
+
g
+
fg
⇒ (2) is correct
1
⇒ I n < I n−2 ⇒ I n < − I n (from (i)) Again given f '( x) g '( x ) = c
n −1
DIferentating both sides w.r.t., we get
1
⇒ 2I n <
(n − 1) f '( x) g "( x) + g '( x) − f '''( x). g ( x) + g ''( x). f '( x ) + f ( x). g '''( x ) + 0

1
= f '''( x). g ( x) + g '''( x). f ( x) + 0 [from (3)]
and I n + 2 < I n ⇒ − In < In Now dividing both sides by F(x) = f(x) g(x)
n +1
Then
1
⇒ < 2I n F '''( x ) f '''( x) g '''( x) F ''' f ''' g '''
n +1 = + or = +
from (1) and (2) we get F ( x) f ( x) g ( x) F f g

PAGE
1 1 ⇒ (3) is correct
< 2I n < ⇒ (3) is correct
(n + 1) (n − 1) 79.a. The equation x 2 + y 2 = 25 represents a circle
77.c. R ∩ R' with centre (0, 0) and radius 5 and the equation
 axeax + e ax ; x ≤ 0 4 2
⇒ f '( x) =  [ f '( x)is continuous at x = 0]
1 + 2ax − 3 x ; x > 0
2 y= x represents a parabola with vertex in
9
and
2ae + a xe ; x ≤ 0
f "( x ) = 
ax 2 ax
www.aieeepage.com
the Figure = {( x, y ) : −3 ≤ x ≤ 3, 0 ≤ y ≤ 5} .
Thus dom R ∩ R ' = [−3, 3] and range
[ f "( x ) is continuous at x = 0]
 2a − 6 x ; x>0
R ∩ R ' = [0, 5] ⊃ [0, 4]
Now f "( x ) > 0 ⇒ 2a − 6 x > 0
(0, 5)
I x > 0 or 2aeax + a 2 xeax > 0 if x ≤ 0 (−3, 4) (3, 4)

a 2
⇒ x< I x > 0 or x > − if x ≤ 0
3 a
(−3, 0) (3, 0)
2 a
⇒ f "( x ) > 0if - < x <
a 3
 2 a Since (0, 0) ∈ R ∩ R ' and (0, 5) ∈ R ∩ R '
∴ f '( x ) increases at x ∈  − ,  ∴ 0 is related to 0 as will as 5
 a 3
Hence R ∩ R ' does not define a function. Also,
78.a. Given F (x) = f(x). g(x)
any line parallel to y-axis contains infinite points
DIferentiating both sides w.r.t x we get
in the region -3 < x < 3
F '( x ) = f '( x ) + g '( x). f ( x )
So, there are infinitely many images for each value
 f ( x) g ( x) 

PAGE
⇒ F '( x) = f '( x) g '( x)  + of x ∈ ( −3, 3), thus R ∩ R ' cannot be a function.

 f '( x) g '( x)  80.b. The equation of the line can be written in the slope
form as
 f g
⇒ F ' = c  +  ⇒ (1) is correct a1 / 3 a
y=− x+
 f ' g ' b 1/ 3
(− a 1/ 3
/ b1/ 3 )
Again dIferentiating both sides sides w.r.t x we get

www.aieeepage.com
1/ 3
a a
i.e.y = mx + where m =
F "( x) = f ''( x).g ( x) + g ''( x). f ( x) + 2 f '( x). g '( x) m b 1/ 3

So it touches the parabola y 2 = 4ax


⇒ F "( x ) = f ''( x).g ( x) + g ''( x). f ( x) + 2c(2..) The equation of the line can also be written in the
Dividing both sides by F ( x ) = f ( x ).g ( x ) form

{∵ f '( x ).g '( x ) = c} b1/ 3 b


x=− y+
F '( x) f "( x ) g "( x) 2c
a 1/ 3
(− b 1/ 3
/ a1 / 3 )
then = + +
F ( x) f ( x) g ( x) f ( x) g ( x)

PAGE - A Premier Institution for IIT - JEE & AIEEE at the National Level 14 of 24
b - b1/3  2 tan θ 
i.e. x = my + where m = 1 / 3 2 
m a 2tan 2θ 1 − tan 2 θ 
⇒ y = tan 4θ = =
So it touches the parabola x2 = 4by also. 1 - tan 2 2θ  2 tan θ 
2
1−  
81.b. The given equation can be written as  1 − tan 2 θ 

( x 2 + y 2 ) (cos2 θ sin 2 α + sin 2θ) 2 × 2 x(1 − x 2 ) 4 x(1 − x 2 )


= 2 2 2
=
2 2
= x tan α - 2xy tan α sin θ + y sin θ 2 2
(1 − x ) − 4 x 1 − 6x2 + x4

or = (cos 2 θ sin 2 α + sin 2 θ − tan 2 α) so that y is finite I x 4 − 6 x 2 + 1 ≠ 0

x 2 + 2(tan α sin θ)xy + cos 2 θ sin 2 αy 2 = 0 6 ± 36 − 4


x2 ≠ ≠ 3± 2 2
Since the slope of these lines are given as 2
tan θ1 and θ 2 . 84.b. Put x = 0 in the given determinant, we get a = 0.
DIferntiating ∆ column by column

PAGE
−2 tan α sinθ
Sum of the slopes = ex sin x 1 ex cos x 1
cos 2 θ sin 2 α
2 2x
∆' = − sin x log e (1 + x) 1 + cos x 1 + 0 = b + 2cx
1 1 + x2
1 x2 1
− 2 tan α × x 2x 1
⇒ tan θ1 + tan θ 2 = 2  π
3 2
∵ θ =  Now, putting x = 0
× sin α  6
4
www.aieeepage.com
1 0 1 1 1 1
8 b= 0 0 1+ 1 0 1 = −1
= − cosec 2αα
3 1 0 1 0 0 1
82.a. P, Q divide AB in the ratio of 1 : 2 and 2 : 1
respectively and hence the coordinates of P and Q 10   10   10 
85.a. p = E 2 (10!) =   +  2  +  3  = 5 + 2 + 1 = 8
 2  2  2 
1× 2 + 2(−3) 1× 3 + 2 × 2 
are  ,  and
 3 3  10   10 
q = E 3 (10!) =   +  2  = 3 + 1 = 4
 3  3 
 2 × 2 + 1(−3) 2 × 3 + 1× 2 
 , 
 3 3  10 
r = E 5 (10!) =   = 2
5
 4 7 1 8 26
⇒ P − ,  and Q ,  and PQ = ...(i)
 3 3   3 3  3 10 
s = E 7 (10!) =   = 1
and QR is perpendicular to PQ. 7
8 8 7 86.a. A.M. of x, z = y, G.M. of x, z = xz and
y− −
⇒ 3× 3 3 = −1  8  1
1 1 4 ⇒  y −  = −5 x −  ..(ii) A.M ≥ G.M. So, y 2 ≥ xz.
x− +  3   3
3 3 3 2xz
Also, AM ≥ HM ⇒ y ≥

PAGE
From (i) and (ii), we get x+z
x+ y x+ y x+ y y+ z y+z
2 = = , =
 1 26 1 1 2 2y − x x + z − x z 2y − z x
26 x −  = , x − = ± ⇒ x = 0 or x = .
 3  9 3 3 3
x+ y y+z
13 +
2y − x 2y − z x+ y y+z
From (ii) when x = 0, y = and when ∴ ≥ .

2
3
www.aieeepage.com 2 z x

x = , y =1 y( x + y + z ) 3y2
3 = 1+ = 1+ [∵ x + z = 2 y]
So the required coordinates of R are xz xz

 13   2  x+ y y+z y2
 0,  or  , 1. ∴ + ≥ 2 1 + 3. ≥4 [∵ y 2 ≥ xz ]
 3  3  2y − x 2y − z xz
83.c. I we put
87.a. Sinc LHS > 0 ⇒ k > 1
x = tan θ, the given equality becomes tan -1 y = 4θ Now 2 log, k - 4 log, 3 = 7

PAGE - A Premier Institution for IIT - JEE & AIEEE at the National Level 15 of 24
Let log k = t > 0 as k > 1. Then, 2t - 4/t = 7 90.c. Since sum of binomial co-efficients in the
expansion of
1
⇒ 2t 2 − 7 k − 4 = 0 ⇒ t = − , 4
2 (5 x / 4 + 35 / 4 ) n = 64
But t > 0, so log 3 k = 4 ⇒ k = 34 = 81 ∴ (1 + n) n = 64 ⇒ 2 n = 64 = 26 ∴ n = 6
Term with the greatest binomial co-efficient
π/4 < tan -1k < π/2 because k > 1.
= (n - 1) + Third term
Also, 0 < cot −1 k < π as k > 3 ⇒ 6C3 (3− x / 4 ) 3 (35 x / 4 ) 3
6
88.b. Since z satisfies = 6 − 1 + 6 C 2 (3− x / 4 ) 4 (35 x / 4 ) 4
| z - 8 - 6i | + | z -14 - 6i | = 10 ....(1) 6
∴ locus of z is an ellipse with foci (8, 60 and ⇒ C3 . 33 x = 5 + 15.33 x / 2
(14, 6) and length of its major axis = 10 Let 3 x / 2 = y ∴ 20 y 2 = 5 + 15 y
(1) can be written as
⇒ 4 y 2 − 3y −1 = 0

PAGE
( x − 1) 2 ( y − 6) 2
+ =1
25 16 ⇒ (4 y + 1) ( y − 1) = 0 ⇒ y = 1

14 + 8 6 + 6 1
[∵ centre of the ellipse , = (11, 6) [∵ y > 0 ∴ y = − is not possible]
2 2 4
2a = 10 ∴a = 5 ∴ 3 x / 2 = 1 = 30 ⇒ x = 0

www.aieeepage.com
2ab = (14 − 8) 2 + (0) 2 = 6 91.c. b 2 − 4ac < 0
For b = 1 any a and c which can be chosen in
3 4 ways
∴10e = 6 ∴ e =
5 For b = 2 either a = 1, c = 2
or a = 2, c = 1
b 2 = a 2 (1 − e 2 ) or a = 2, c = 2
 9  ⇒ Required probability = 7/8
= 251 −  = 16] ∞ ∞
 25  2n 2n
92.a. ab = Σ Σ
n =1 ( 2n − 1)! n =1 ( 2n + 1)!
⇒ 16 x 2 + 25 y 2 − 352 x − 300 y + 2436 = 0 ...(2)
Pair of tangents to (2) from the origin can be ∞ 2n − 1 + 1 ∞ 2n + 1 − 1
= Σ Σ
found as n =1 ( 2n − 1)! n =1 ( 2n + 1)!

10 x 2 − 66 xy + 48 y 2 = 0 ...(3)
 ∞ 1 1 
Max, and Min. arguments of z are slopes of the =  Σ  +
n =1 (2n − 2)! (2n − 1)! 
lines given by (3)  

33 + 604  ∞ 1 1 
∴ Max. Arg. z = tan -1 =  Σ  −  
48 
 n=1 (2n)! (2n + 1)!  
89.b. Let the three numbers in G.P. be a, ar, ar 2

PAGE
 1  1 1   1 1  
∴ a(1 + r + r 2 ) = 14 ...(1) =  1 +  +  +  +  +  + ..
  1!   2! 3!   4! 5!  
2
Again a + 1, ar + 1, ar − 1 are in A.P.
 1 1   1 1  
∴ a (r 2 + 1) = 2 (ar + 1) ...(2) =   −  +  −  + ..
  2! 3!   4! 5!  
r 2 + r +1 14
⇒ 2
= =7  1 1 1 
www.aieeepage.com
r − 2r + 1 2 = 1 + + + + ....
 1! 2! 3! 
⇒ 6r 2 − 15r + 16 = 9 ⇒ 2r 2 − 5r + 2 = 0
 1 1 1 1 
1 =  (1 − 1) +  −  +  −  + ....
⇒ (r − 2) (2r − 1) = 0 ⇒ r = 2 or   2! 3!   4! 5!  
2
1 1
I r = 2, a = 2 and I r = ,a=8 = (e) (e −1 ) = e. = 1
2 e
∴ terms are 2, 4, and 8, 4, 2. Hence lowest of the Hence ab = 1
original terms = 2

PAGE - A Premier Institution for IIT - JEE & AIEEE at the National Level 16 of 24
93.d. Let f ( x) = px 2 + qx + r 1 1 2 2
⇒ . | AB × AC | p =
∴ f ( a + h) = p ( a + h) + q ( a + h ) + r2 3 2 3
1
f (a) = pa 2 + qa + r ⇒ | (i × k ) × ( j + k − 2 i ) p = 2 2
2
f (a + h) − f (a ) p (h 2 + 2ah) + qh ⇒ p=2
∴ =
a+h−a h ∴ Position vector of E, I E divides median AF

 h in the ratio λ : 1 .
= p ( h + 2a ) + q = 2 p a +  + q
 2 λ λ
Now AE = AF ; = (i − j − k )
λ +1 λ +1
 h
= f ' a +  2
 2  λ 
⇒ AE 2 =   − 3 But p 2 + AE 2 = AD2 ;
h  λ +1

PAGE
Hence c = a + which is independent of p, q, r.
2
a ∴ (2) 2 + AE 2 = (4) 2 ⇒ AE 2 = 16 − 4 = 12
dx
94.b. Let I = ∫ 1+ e
0
f ( x) 97.d. Operate C1 + C 2 + C 3 the given determinant
has first column as
a a
dx dx x y z
Also I = ∫ 1+ e = ∫ 1+ e
0
f (a − x)
0 www.aieeepage.com
−− f ( x) loga  . .  i.e. loga 1 i.e. 0
 y z x
a a
e f ( x) 1 + e f ( x)  y z x
= ∫ 1+ e f ( x)
dx ; ∴ 2I = ∫ 1+ e f ( x)
dx = a logb  . .  i.e. logb 1 i.e. 0
0 0  z x y
a  z x y
∴ I=
2 logc  . .  i.e. logc 1 i.e. 0
x y z
dy dz
95.d. Put x + y = z ∴1 + = ∴ given determinant = 0.
dx dx 98.b. The equation of the given coaxal system is
dz x2 + y 2 − 2αx-2 βy + c + λ(x 2 + y2 ) = 0 or
∴ − 1 = sin z + cos z
dx
2α 2β c
dz x2 + y2 − x− y+ =0
⇒ = 1 + sin z + cos z 1+ λ 1+ λ 1+ λ
dx Its centre is
dz
⇒ = dx  α β  α 2 + β 2 − c(1 + λ)
1 + sin z + cos z  ,  amd radius is
 1+ λ 1+ λ  |1+ λ |
dz
⇒ = dx

PAGE
z z z α2 + β2
2 cos 2 + 2 sin cos The radius vanishes I 1 + λ =
2 2 2 c

1 z  cα cβ 
sec 2 dz So, the other limiting point is  2 2
, 2 2 
⇒ ∫ 2
1 + tan
2
z
= dx + c ∫  α +β α +β 

2 99.a. For any value of λ, the given equation represents

⇒ log 1 + tan
z
= x+c
www.aieeepage.com
the circle x + y
circles passing through points of intersection of
2 2
+ 2ax + 2by + c = 0 and the line
2
ax - by + 1 = 0. So, ax - by + 1 = 0 is the common
 x+ y chord for each circle of the system, henc it is the
⇒ log 1 + tan   = x+c radical axis.
 2  100.c. The line of centres will be perpendicular to radical
axis, so its equation is bx + ay + k = 0
1 2 2 It must pass through the centre (-a, -b) of the
96.c. Volume of tetrahedron = (∆ ABC) p =
3 3 memeber of the system, so, k = 2ab
Hence the desired equationis bx + ay + 2ab = 0

PAGE - A Premier Institution for IIT - JEE & AIEEE at the National Level 17 of 24
Paper - I : Chemistry - 1 to 50
Solutions

101.d. All of the choices are correct comes sp 2 hybridized again. The three groups attached
Pb ++
+ 2HCl 
→ PbCl 2 ↓ 
→ PbS ↓
H 2S
to the carbonyl carbon ar (1) the oxygen atom with which
102.d. White ppt Black ppt
it forms a double bond, (2) an OH group, and (3) the R
dissolves on boiling
group.
103.a. N 2 H 4 
→ N2 In this case, the R group is a methyl group, and so the
0
molecule left behind after the alcohol departs is
130 CH 3COOH. This is acetic acid.
Equivalent weight of N 2 H 6SO 4 = = 32.5
4
Number of equivalents of 107.b. Step 1 is an S N 2 reaction and is thus favored by a polar
5 1 aprotic solvent like DMF. N-methylformamide is a polar
KMnO 4 = 20 × =

PAGE
protic solvent. (Note that there is a hydrogen bonded to
50 × 1000 500 a nitrogen atom. The molecule can thus participate in
and I weight of hydraize sulphate be x gm then hydrogen bonding). It will solvate the nucleophile and
reduce the rate of the reaction.
x
equivalents of hydrazine sulphate = 108.b. Tertiary alkyl halides will not be able to undergo the SN2
32.5 reaction outlined in the passage because of steric hin-
drance (eliminate choice A). However, they can undergo
1 x 32.5
∴ = or x = = 0.065 g an SN1 reaction with water (Choic B) to make a stable

www.aieeepage.com
500 32.5 500 tertiary carbocation, which can accept water as a nucleo-
phile.
Hence wt of N 2 H 6SO 4 in 10 ml solution = 0.065 g
Choice C makes a tertiary Griganard reagent, a BASIC
∵ wt of N 2 H 6SO 4 in 1000 ml solution = 65 g carbon reagent, which picks up a PROTON form water,
104.b. Only cis isomer can easily form a chelate ring with not a hydroxyl group!
oxalate group ; the trans isomer can not form a Choice D, as alluded to in the passage, gives mainly E2
chelate ring . Both are square planar. (alkene) products, especially in this case since the halide
is hindered.
105. a. In the IUPAC system, esters are named as alkyl
alkanoates, The “alkyl” group is the one that is attached 109.a. The saponIication mechanism (outlined below) is required
to the oxygen atom, while the “alkan-” part contains the to understand this question. The bottom line is that the
the carbonyl carbon : nucleophilic hydroxide ion attacks the ester carbonly (not
the secondary carbon in the alcohol product). So the iso-
O topic lable ends up still attached to the alcohol product,
but not to the carboxylic acid.
Choice D would be the correct answer I the isotopic lable
O C CH 3 were in the attacking hydroxide and the ester was
unlabelled.
ethanoate
butyl O
O OH
O
Choice b is incorrect because it would have an ehtyl group O
H3C O
18 O18 18
O
attached to the oxygen atom : H3C
O
proton products

PAGE
NaOH +
O HzO HO CH3 exchange
(Choice A)

O C
110.a Hydrated ferric chloride (FeCl3 .6H 2 O) upon
ethyl butanoate heating gets hydrolysed by its own molecules of
water of crystallisation to give Fe(OH) 3 , which
Choice c is incorrect because acetate is the common name

www.aieeepage.com
for ethanoate. Ethyl acetate is ethyl ethanoate which is changes to Fe 2 O3 of on further heating
not the ester formed in Ste 1. (it would not have been the
IUPAC name anyway) Fe Cl .6 H O 
→ Fe(OH) + 3HCl + 3H O
2 3 2 3 2

Choice d is incorrect because carboxylate is the salt of a 2Fe(OH) 3 


→ Fe 2 O 3 
→ Fe 2 O 3 +3H 2 O
carboxylic aicd.
106.c. Acid-catalysed hydrolysis of the ester ROOCR’ proceeds -21.76 × 10 -19
via the formation of a tetrahedral intermediate. The OR’ 111.a E n of H = J
n2
group is then protonated and departs as the alcohol R’OH
as the carbonyl bodn reforms and the carbon atom be- -21.76 × 10-19
∴ E n of He+ = 2
× Z2 J
n

PAGE - A Premier Institution for IIT - JEE & AIEEE at the National Level 18 of1 of
24 7
distance travelled by the gas .Hence
-21.76 × 10-19 × 4
∴ +
E 3of He = J kP
9 r1 (of HCl gas ) at pressure P = 60=
36.5
Hence energy equivalent to E3 must be supplied
to remove the electron from 3rd orbit of and r2 (of NH 3 ) at 1 atm. pressure

He+ .Wavelength corresponding to this energy can k ×1


P = 40 =
hc 17
be determined by applying the relation E =
λ
r1 60 kP 17
= = ×
hc 6.625 × 10−34 × 3 × 108 × 9 r2 40 36.5 k × 1
or λ= =
E 21.76 × 10−19 × 4
= 2055 × 10-10 m = 2500Å
60 36.5
P= × = 2.197atm
112.b. CO & N2can be separated by bubbbbbling through 40 17
ammonical cuprous chloride in which CO is

PAGE
118.a. Hydrometallurgy :
absorbed but not N2 .N2 O2can be seperated by
CuCO 3 .Cu(OH) 2 
Roasting
→ 2CuO+H 2 O+CO
bubbling through alkaline solution of pyrogallol in Malachite
which oxygen is absorbed but not
CuO.H 2 SO4 
Dissolution
→ CuSO 4 +H 2 O;
N 2 .NH3 & N 2 can be seperated by bubbbling (Soluble)

thropugh H 2SO 4 when NH3 is absorbed but not CuSO 4 + Fe 


→ Cu + FeSO 4
ppt Soluble

www.aieeepage.com
N 2 .Ammonia is recovered by heating the solution
More electropositive metal replaces lesser one
of (NH 4 )2 SO4 with excess of NaOH
Fe O Smelting : 2 3 + 3C 
→ 2Fe + 3CO
Reduction with C
PH 3 & NH 3 can be seperated by bubbling
Self reduction :
through water in which NH 3 dissolved but not
2Cu 2 S + 3O2 
→ 2 Cu 2 O + 2 SO2 ;
PH 3 .
Cu 2 S + 2Cu2 O 
→ 6Cu + SO2
113.d. All the choices are correct
114.a. Dipole moment µ = e × d coulomb metre Electrolytic Reduction : Electrolysis of Al2 O3
in molten cryolite
For KCl d = 2.6 ×10−10 m For complete separation 119.b. A positive Beilstein’s test for halogen does not
of unit charge always indicate the presence of halogen since
( electronic charge ) (e) = 1.602 × 10 −19 C some halogen free compounds viz . urea ,
thiourea, amides etc. also respond this test . The
Hence µ = 1.602 × 10 −19 × 2.6 × 10 −10 reason being the fact that these halogen free
= 4.1652 ×10−29 compounds form cuprous cyanide which is
volatile and decomposes to copper which burns
µ KCl = 3.336 × 10 −29 m with green flame .
120.a. Initial concentration of each gas = 1 mole
3.336 × 10-29
∴ % ionic character of KCl = = 80.09 % Let the No .of moles of NO2 reacted at
4.1652 × 10−29 × 100
equilibrium = x , Then

PAGE
115.a. Both lattice energy and hydrolysis energy decrease
s moving down the group due to gradual increase SO2 ( g ) + NO2 ( g ) SO3 ( g ) + NO2 ( g )
in size of M ++ ion .So former tends to increase the ( 1- x ) ( 1- x ) ( 1+ x ) (1+x)
solubility while latter tends to decrease it . But lattice Now we know that
energy has dominating role here .And therfore ,
solubility increase down the group . [ SO3 ][ NO ] (1 + x)(1 + x)
116.c. EA of representative elements increases from left , [ SO ] [ NO ] = K c or
(1 − x)(1 − x)
= 16

www.aieeepage.com
2 2
to right in a period ( barring few exceptions ) and
decreases down the group . However ,the second (1 + x) 1+ x 2
= 16 or =4
period group members ( e . g N , O , F ) ,being small (1 − x) 2 1− x
in incoming electron and the the other electorns in
the atom . 3
1 + x = 4 - 4x or = 0.6
5x = 3 x=
117.a. Since the pressures of gases are dIferent ,and the 5
temperature is constant , the rate at which Thus the concentration of No at equilibrium
molecules of the two gases dIfuse is directly = 1 + x = 1 -0.6 = 0.4 moles
proportional to the pressure . This rate of
dIfusion is also directly proportional to the

PAGE - A Premier Institution for IIT - JEE & AIEEE at the National Level 19 of 24
Heat of dissociation per mole =
O
|| 30
= 2000 cal = 2.0k cal
121.c. a) R - C - NH 2 
Br2 / NaOH
→ R − NH 2 0.015
(Hofmann reaction) 126.c. In vapour phase 1 mole (or 78 g) benzene has volume

O 0 87 ×1
at 20 C = × 2750 mL
|| 0.877
b) R - C - N 3  
heat
→ R − NH 2 (Curtius reaction) 1 mole (or 92g ) toluene has volume at 20°C =
92 × 1
O × 7720 mL
0.867
||
c) R - C - OH+HN 3 
H 2 SO4
→ R − NH 2 PBo 78 × 2750
∴ × = 1 × 0.0821 × 293
( Schmidt reaction ) 760 0.877 × 1000

PAGE
O or PBo = 74.74cm
|| Similarly,
-
d) R - C - NHOH 
HO
→ R − NH 2 PTo 92 × 7720
× = 1 × 0.821 × 293
( Lossen reaction ) 760 0.867 × 1000
122.a. At pH < isoelectric pH, -NH 2 group is protonated
or PTo = 22.37 mm;
+
www.aieeepage.com
to - N H3 and the sol is positively charged. At
127d. Rate = k[A] [ B] x y
pH > isoelectric pH, -COOH is deprotonated to
-COO- and the sol is negatively charged. At (rate)2 [0.024] x [0.070] y 8 ×10−3
isoelectric pH both groups are equally ionised, the =k = =8
sol particles carry no net charge.
(rate)3 [0.024] x [0.035] y 1×10−3
123.a. Standard enthalpy of hydrogenation of cyclohexene
(-199kj mol-1) means the enthalpy of hydrogenation
or 2 y = 8 23 , y = 3
of one double bond. Now benzene has three double
bonds, the enthalpy of the reaction would be = 3 x (rate) 2 8 ×10−3 k[0.024] x [0.070] y
= = = 2x
-119 = - 119 = - 357 kj mol-1. (rate)1 8 ×10−3 k[0.012] x [0.070] y
+ 3 H2 d [ A]
or, − = k [ B ]3
dt
Actual enthalpy of the reaction can be evaluated
as below: 2AB2 → 2AB + B2
128.d.
∆H (Reaction) = ∆H of (Product) − ∆H of (Reactants)
1− α α α/2
= -156 - (49+0) α 2+α
= 205 kj mol-1 Totalmoles = 1 - α + α + =
2 2

PAGE
∴ Resonance energy =
(1 − α)p 2(1 − α)p
∆H Exp − ∆ cal = −357 − (−205) = −152kj mol −1 PAB2 = =
(2 + α)/2 2+α
124.d. All of the choices, since
1 αp 2αp
Q=S+ D + I − E −U PAB = =
2 (2 + α)/2 2 + α

125.d. Moles of the acid neutralized =


200
1000
www.aieeepage.com
× 0.1 = 0.02 α
p
αp
PB2 = 2 =
Moles of acid remaining undissociated =
(2 + α) 2 + α
 25  2
0.021 −  = 0.015
 100  HENCE,
Heat evolved during neutralization for 100%
ionization = 13700 × 0.02 = 274cal
Heat used up for the dissociation of 0.015 mole
= 274 - 244 = 30 cal

PAGE - A Premier Institution for IIT - JEE & AIEEE at the National Level 20 of 24
2 10-17 10 -17
 2αp  αp = c.c. = c.c
  × 3.012 × 136 409632
p 2AB pB2 2+α 2+α α p
3
kP = = = (α << 1)
2 2 2 1000000 × 10 −23
p AB2  2(1 − α ) p  c.c. = 2.44 × 10 − 23 c.c.
 2 + x  409632
Since each mercury atom occupies a cube of edge
129.c. As for emission of one α - particle atomic mass length equal to its diameter, therefore diameter of
one Hg atom
decreases by 4 and number by 2. Further for the
1 1
emission of one β - particle, the atomic mass do − 23 3 − 24 3
= (2.44 × 10 ) cm = (24.4 × 10 ) cm.
not change but the atomic number increases by 1.
So we first find the α - particles; = 2.905 × 10 −8 cm ≅ 2.91Å cm.
Decreases in atomic mass = 234 - 206 = 28 133.c.

PAGE
28
No. of α - particles emitted = =7 I. [ Ag + ]. = K sp ( AgCl ) = 1.0 × 10 − 10 = 1.0 × 10 − 5 M
4
Hence, atomic number decreases to
II.  Ag +  = K sp ( AgI ) = 1.0 × 10 −16 = 1.0 × 10 −8 M
90 − (7 × 2) = 76
Now, atomic number of pb = 82, which is more III. Ag ( NH 3 ) 2+ Ag + + 2 NH 3 ;
by (82 - 76) = 6
 Ag +  [ NH 3 ] [ Ag + ] × 0.12
2

K d = 1.0 × 10−8 = 
www.aieeepage.com
Therefore, β - particles emitted = 6 = ⇒ [ Ag + ] = 1.0 ×10−6 M
 Ag ( NH 3 )2+  1.0
130.b. Activated
complex
D+E+F iv. Ag (CN ) −2 
→ Ag + + 2CN − ,
[ Ag + ][CN − ]2 [ Ag + ] × 0.12
Enthalpy

C
B+ K d = 1.0 × 10 − 21 = = = 1.0 × 10 − 19 M
A+ [ Ag ( CN ) 2− ] 1.0

Reaction coordinate
134.a. Density of 39% H 2 SO4 before discharging =
In the above reaction, heat is gained, thus the
1.294 g/ml
reaction is endothermic
131.b. (i) The product may be meso or achiral ∴ Amount of H 2 SO4 in the solution before
(ii) In case an optically active chiraL reagent,
3500 × 1.294 × 39
solvent, or catalyst such as enzyme is used, the discharging = = 1766.3 g
productwill likely to be optically acitve. 100
(iii) The configuration can be changed only by Density of 20% H 2 SO4 after discharging = 1.139
breaking a bond. g/ml
(iv) Changing the configuration at one of the chiral
carbons convetrs one diastereomer to the other. ∴ Amount of H 2 SO4 in the solution after
(v) The terms D and L do not refer to the sign of 3500 ×1.139 × 20
rotation. They refer to the configuration of as discharging = 797.3 g
100

PAGE
stereoisomer relative to that of D-glyceraldehyde.
Amount of H 2 SO4 consumed by the battery
132.d. Avogadro’s number = 6.023× 1013
At. wt. of mercury (Hg) = 200 969
=1766.3-797.3=969.0 g = = 9.887 moles
∵ In 1 g of Hg, th total number of atom = 98
The charging and discharging reactions are
6.023 × 1013 6.023 × 1013
= Pb + SO42 − 
→ PbSO4 + 2e − (charging)
2 × 10 2
www.aieeepage.com
200
+ 2− −
21 21
PbO + 4 H + SO + 2e 
2 → PbSO + 2 H O (discharging)
4 4 2
= 3.0115 × 10 = 3.012 × 10
Density of Mercury (Hg) = 13.6 g/c.c Adding the above two reactions, we get
∴ Volume of 1 atom of mercury (Hg) =
Pb + PbO2 + 2 H 2 SO4 →
disch arg ing
2 PbSO4 + 2 H 2 O
ch arg ing
1 10 ×10 3
c.c. = c.c Thus w see that 2 moles of H 2 SO4 are consumed
3.012 ×1021 ×13.6 3012 ×1021 ×136
to give 2 moles of electrons, i.e., 2 Faraday of
current.

PAGE - A Premier Institution for IIT - JEE & AIEEE at the National Level 21 of 24
1 mole of (H 2 C = CH 2 ) will be the major product (Hofmann
H 2 SO4 = 1 F of H 2 SO4 = 96500 coulombs product or Hofmann elimination)
∴ No. of ampere-hours for which battery is used = β1 (i) CH 31 + H 2C = CH 2
β2 +
96500 × 9.887 β2 (ii) Ag 2O
(from β1 ) (from β 2 ) (from β 3 )
= 265.02 ampere hours NHCH 2 CH 3 (iii) heat
60 × 60
In Hofmann elimination H atom (in the form of
+ +
135.b. 1) Cell reaction : 2H (aq) → H 2(g ) + Zn (2aq
+ Zn (s)  ) proton) is eliminated from that β carbon atom which
has maximum number of H atoms(s). This is due to
PH 2 × [ Zn 2+ ] 1× 0.01
Re action quotient, Q = = = 1, log Q = 0 the fact that Hofmann elimination (an example of E2
[ H + ]2 0.12
reaction) requires anti-coplanar arrangement of the
2) Cell reaction : 2Ag +(aq) + Cu( s ) 
→ Cu(2aq+ ) + 2 Ag ( s ) β H atom and ther leaving group. Further, higher
the changes for this arrangement to remain in
staggered conformation more will be the case of
PH 2 [Cd 2+ ] 1 × 0.01 elimination. Thus greater the number of H atom at
Q= = =1

PAGE
+ 2
[H ] 0.12 β position, higher will be the chances for Me3N+_
PH2 ×[Cd 2+ ] 1× 0.01
and H atom to remain in this particular conformation
3) Cell reaction : 2H +
(aq) → H 2 + Cd(2aq+ ) ; Q =
+ Cd( s ) 
[ H + ]2
=
0.12
=1 (anti - coplanar arrangement of Me3N+- and H atom
PH 2 × [ Zn 2+ ]
in staggered conformation).
1 × 01
4) Cell reaction : 2H (aq)
+
→ H 2 + Zn 2(aq ) ; Q =
+ Zn (s ) 
[H + ]2
=
0.12
= 10 139.c. In case of exothermic dissolution, the solubility of
the solid increases on lowering the temperature.
136.c. MnO −4 + C 2 O 24− + H + 
→ Mn 2+ + CO 2 + H 2 O On cooling, the solution becomes unsaturated and

Equivalent mass of KMnO 4 =


www.aieeepage.com
mol.mass
solid solute does not separate. At 00C, water in the
solution does not freeze.
5
140.b. rCs + / rCI − = 0.90Å ⇒ rCs + = 0.90 × rCI − = 0.90 × 1.80 = 1.62 Å
(‘n’ factor = 7-2 = 5)
Equivalent mass of rCs +
Formula mass 88 For precise fitting of the cubic voids, r must
C 2 O 24− = = = 44 Cl −
2 2
[‘n’ factor = 2(4-3)=2] be 0.732
Which is smaller than the given value of 0.90.
Equivalents of C 2 O 24 − = Equivalents of
Hence, cubic packing of Cl− ions is loose to some
MnO −4 = 30 × 0.05 × 5 ×10 −3
= 7.5 ×10 −3
extent.
Mass of C 2 O 24 − = 7.5 × 10 −3 × 44 = 0.33 g; The Cs + ion at the centre of the body is in touch

0.33 × 100 with 2Cl− ions at the opposite corners of cube.


% C 2 O 24 − = = 66
0 .5 Hence distance between Cs + and Cl −
137.a. Compound (A) on treatment with AgNO3 give white
precipitate of AgCi, which is readily soluble in dil.aq. = rCs + rCl − = 1.62Å + 1.80Å = 3.42Å
NH3. Therefore it has at least one CI_ ion in the
141.a. The four compounds dIfer in two respects :
ionization sphere furthermore chromium has

PAGE
Compounds II has sp3 hybridised N, compounds I
coordination number equal to 6.So its formula is
and III have sp 2 while compound IV has sp
[Cr ( NH 3 ) 4 BrCI]CI hybridised N. Now we know that greater the s
Compound (B) on treatment with AgNO3 gives pale character of an orbital, more tightly its electronsare
yellow precipitate of AgBr soluble in conc. NH3. held and hence lesser will be their availability for
Therefore it has Br − in the ionization sphere.So its protonation causing weak basic character. Thus the
basic character of the three N’s is
formula is [Cr ( NH 3 ) 4 CI 2 ]Br

x
x
x
x
x
x
www.aieeepage.com
x
x
sp N > sp > N > spN . However, in pyrrole
x
x
x
3

electrons on N are part of aromatic sextet, i.e., these


x
2

Cr 3+ ( Z = 24) are delocalised and hence lesser available for


3d 4s 4p
protonation and thus pyrrole is a weaker base than
State of hybridization of chromium in both (a) and pyridine because in pyridine in nonbonding
(B) is d2sp3. Spin magnetic moment of (a) or (B) electrons present in sp2 orbital do not form a part of
µ spin = n (n _ 2) = 3(3 + 2) = 15 = 3.87 BM aromatic sextet. Thus the basic character should
be in the following order.
138.c. The given amine has three β − hydrogens, hence it
can form three alkenes of which least substituted

PAGE - A Premier Institution for IIT - JEE & AIEEE at the National Level 22 of 24
filled five hybrid orbitals form σ - bonds with 4F
H 3C − C ≡ N < N
or atoms and O atom. The pure 5d orbital forms
H
π - bond with 2p orbital of O-atom.
IV III
(sp hybridised N)
2
(sp Hybridised N but
144.a. Although ordinarily chlorine present on benzene
electron pair forms part of sextet) nucleus is dIficult to be replaced, however I it has
electron-withdrawing group in the o- and ρ -
position it is easily replaced. In the present case CI
O
< O < (electronegative group) present on C2 and C4 are
N N N
H H ortho to CI present on C1, hence the latter can be
I II
2
(sp Hybridised N; 3 replaced by OH − . Note that other two chlorines
(sp Hybridised N)
electron pair not a part (at C2 and C4) are meta to one chlorine, so these are
of sextet)
not replaced.
142.d. The product, 2-bromo-4 methylhexane, has two
chirality centers CI OH
1 CI CI

PAGE
2 _

CH 3 Br aq.OH

CH 3CH 2 * CHCH 2 * CHCH 3 4

Since C4 of the product has R configuration in the


CI CI
reactant, and since this chirality center is unaffected hc 6.63 × 10 −34 × 3 × 108
145.d. Energy for photon E = hν = = J = 66.3 × 10 −19 J
by the reaction, its configuration us unaffected λ 300 × 10 −10

www.aieeepage.com
(assuming that the relative priorities of the four K.E. acquired by the electron
group are not changed by the reaction); i.e., the
configuration at C4 in the product is also R.
Addition of HBr involves the formation of = 6.63 × 10−19 − 1× 1.6 × 10−19 = 64.7 × 10−19 J
carbocation, it is attacked by Br − form top as well 1 1
mv 2 = × 9.1× 10 −31 × v 2 ⇒ v = 3.8 × 10 6 ms −1
as bottom face leading to configuration at C2 as R 2 2
as well as S. However, the carbocation does not
146.a. Equation 1 shows the exchange of a sulfide ion for
have a plane of symmetry, it is chiral because of the
two acetate ions. Since sulfide is in a-2 oxidation state
chirality center at C4, hence it will not be attacked
on both sides of the equation, we can eliminate choice
equally from the two faces, leading to a mixture of
(C), which suggests a change in oxidation state. Since
2R and 2S products in a ratio other than 50 : 50.
exchange reaction is not one of the choices, we must
Thus the net result is the formation of two
determine which of the remaining choices is correct.
diastereomeric products [2R, 4R] - and [2S, 4R]- in
Choice (A) works because sulfide is a lewis base that
unequal amounts.
attacks the positively charged lead ion choice (B) is
143.c. a) The S-atom in the excited state (3s 2 3p 3 3d1 ) has incorrect, an Arrhenius acid base reaction would
2
involve hydrogen ions. Choice (D) is incorrect
sp3 hybridization - (sp 3 ) (sp 3 )1 (sp 3 )1 3d1. The because no neutralization is taking place.
three hybrid orbitals form three (S-O) σ − bonds
147.b. The solubility of calcium sulfide and magnesium
and the 3d pure orbital gives (S-O) π − bond . sulfide is very low, so these solides will form readily
from solution. When hard metal ions are present in

PAGE
b) In PO34− , P-atom in its excited state (3s1sp 3 3d1 )
the water used to rinse the hair before dye application,
is sp3 hybridized. Four hybrid orbitals forms (P-O) they react with the sulfide in hair keratin before lead
and bismuth ions have a chance to attack. Therefore,
σ - bonds whereas 3d1 gives (P-O) π - bond . choice (B) is correct. Choice (A) is incorrect because
c) In NO 3- , N-atom is sp 2 hybridized as citrate and acetate ions will not precipitate with hard
metal ions - these will remain in solution. Choice (C) is
(sp 2 ) 2 (sp 2 )1 (sp 2 )1 2p1z . incorrect because in solution, additional ions will not

Completely filled sp 2 hybrid orbital on N-atom www.aieeepage.com decrease the effectiveness of other ions through
collision reduction. I this wee the case, the hair dye
donates an electron pair to the vacant 2p orbital of could be left on the hair longer and it would have time
O atom to form dative bond. Other two hybrid to react fully. Choice (D) is incorrect because calcium
and magnesium do not precipitate in elemental form.
orbitals form (N-O) σ - bonds . The pure 2p1z orbital
They are much more stable as positively charged ions-
forms (N-O) π - bond . remember that they are alkaline earth metals, which
d) In XeOF 4 , X e -atom in its excited state prefer to be in a + 2 oxidation state.
(5s 2 5 p3 5d 3 ) is sp 3d 2 hybridized. The singly

PAGE - A Premier Institution for IIT - JEE & AIEEE at the National Level 23 of 24
148.c. This is a thinking question - you can’t just search for the 150.a. There are three crucial things to remember when
answer in the passage prose or equations. The TEST attacking an equilibrium expression question.
loves to throw a few of these at you - after all, they are
trying to test your critical thinking! Let’s examine how * Products appear in the numerator reactants in
we can attack this question effectively. Bismuth is in the the denominator
p-block right next to lead. Before it loses any electrons, it
has a half-full p subshell. Since it is so far down on the * A coefficient in the reaction equation corresponds
periodic table, it is considered a metal, not a metalloid. to a superscript in the equilibrium expression
Theefore, you can eliminate choice (B). The question is
basically asking you why bismuty loses p electrons but * Pure solids and liquids do not appear in the
keeps s electrons. Choice (A) doesn’t give an answer to equilibrium expression.
that-it suggests that the opposite is true by mentioning
the noble gas configuration bismuthy could adopt by los-
ing all of its valence electrons. Choice (D) anasers a dIfer-
ent question about gaining electrons, so it must be incorect.
This leaves only choice (C), which correctly explains that
the s orbital for bismuth is much lower in energy, so the

PAGE
element is more stable withou losing those electrons.
149.d. This is another le chatelier’s principle question. The
simplest thing to do is to consider how the
equilibrium will be affected, then attack the first
part of the statements regarding the effectiveness
of the hair dye. Adding more acetate ions will shIt
the reaction to the left since there will be more

www.aieeepage.com
products in solution. Therefore, we can eliminate
choices (A) and (B). Now, how will this affect the
effectiveness of the hair dye? It will decrease, since
less lead sulfide will be produced. Therefore, choice
(D) is correct.

PAGE
www.aieeepage.com

PAGE - A Premier Institution for IIT - JEE & AIEEE at the National Level 24 of 24

You might also like